Difference between revisions of "2004 AMC 12A Problems/Problem 7"

m (fixed typo in answer choice)
(Solution 1)
(10 intermediate revisions by 7 users not shown)
Line 1: Line 1:
 +
{{duplicate|[[2004 AMC 12A Problems|2004 AMC 12A #7]] and [[2004 AMC 10A Problems/Problem 8|2004 AMC 10A #8]]}}
 
==Problem==
 
==Problem==
A game is played with tokens according to the following rule.  In each round, the player with the most tokens gives one token to each of the other players and also places one token in the discard pile.  The game ends when some player runs out of tokens.  Players <math>A</math>, <math>B</math>, and <math>C</math> start with 15, 14, and 13 tokens, respectively.  How many rounds will there be in the game?
+
A game is played with tokens according to the following rule.  In each round, the player with the most tokens gives one token to each of the other players and also places one token in the discard pile.  The game ends when some player runs out of tokens.  Players <math>A</math>, <math>B</math>, and <math>C</math> start with <math>15</math>, <math>14</math>, and <math>13</math> tokens, respectively.  How many rounds will there be in the game?
  
 
<math> \mathrm{(A) \ } 36 \qquad \mathrm{(B) \ } 37 \qquad \mathrm{(C) \ } 38 \qquad \mathrm{(D) \ } 39 \qquad \mathrm{(E) \ } 40  </math>
 
<math> \mathrm{(A) \ } 36 \qquad \mathrm{(B) \ } 37 \qquad \mathrm{(C) \ } 38 \qquad \mathrm{(D) \ } 39 \qquad \mathrm{(E) \ } 40  </math>
  
==Solution==
+
==Solutions==
Look at a set of 3 rounds, where the players have <math>x+1</math>, <math>x</math>, and <math>x-1</math> tokens.  Each of the players will gain two tokens from the others and give away 3 tokens, so overall, each player will lose 1 token.
 
  
Therefore, after 12 sets of 3 rounds, or 36 rounds, the players will have 3, 2, and 1 tokens, repectively.  After 1 more round, player <math>A</math> will give away his last 3 tokens and the game will stop <math>\Rightarrow\mathrm{(B)}</math>.
+
===Solution 1===
 +
We look at a set of three rounds, where the players begin with <math>x+1</math>, <math>x</math>, and <math>x-1</math> tokens.
 +
After three rounds, there will be a net loss of <math>1</math> token per player (they receive two tokens and lose three). Therefore, after <math>36</math> rounds -- or <math>12</math> three-round sets, <math>A,B</math> and <math>C</math> will have <math>3</math>, <math>2</math>, and <math>1</math> tokens, respectively.  After <math>1</math> more round, player <math>A</math> will give away <math>3</math> tokens, leaving them empty-handed, and thus the game will end. We then have there are <math>36+1=\boxed{\mathrm{(B)}\ 37}</math> rounds until the game ends.
  
==See Also==
+
===Solution 2===
  
*[[2004 AMC 10A Problems]]
+
Let's bash a few rounds. The amounts are for players <math>1,2,</math> and <math>3</math>, respectively.
  
*[[2004 AMC 10A Problems/Problem 7|Previous Problem]]
+
First round: <math>15,14,13</math> (given)
 +
Second round: <math>12,15,14</math>
 +
Third round: <math>13,12,15</math>
 +
Fourth round: <math>14,13,12</math>
  
*[[2004 AMC 10A Problems/Problem 9|Next Problem]]
+
We see that after <math>3</math> rounds are played, we have the exact same scenario as the first round but with one token less per player. So, the sequence <math>1,4,7,10...</math> where each of the next members are <math>3</math> greater than the previous one corresponds with the sequence <math>15,14,13,12...</math> where the first sequence represents the round and the second sequence represents the number of tokens player <math>1</math> has. But we note that once player <math>1</math> reaches <math>3</math> coins, the game will end on his next turn as he must give away all his coins. Therefore, we want the <math>15-3+1=13</math>th number in the sequence <math>1,4,7,10...</math> which is <math>\boxed{\mathrm{(B)}\ 37}</math>.
 +
 
 +
Solution by franzliszt
 +
 
 +
== See also ==
 +
{{AMC12 box|year=2004|ab=A|num-b=6|num-a=8}}
 +
{{AMC10 box|year=2004|ab=A|num-b=7|num-a=9}}
 +
{{MAA Notice}}

Revision as of 21:31, 16 July 2020

The following problem is from both the 2004 AMC 12A #7 and 2004 AMC 10A #8, so both problems redirect to this page.

Problem

A game is played with tokens according to the following rule. In each round, the player with the most tokens gives one token to each of the other players and also places one token in the discard pile. The game ends when some player runs out of tokens. Players $A$, $B$, and $C$ start with $15$, $14$, and $13$ tokens, respectively. How many rounds will there be in the game?

$\mathrm{(A) \ } 36 \qquad \mathrm{(B) \ } 37 \qquad \mathrm{(C) \ } 38 \qquad \mathrm{(D) \ } 39 \qquad \mathrm{(E) \ } 40$

Solutions

Solution 1

We look at a set of three rounds, where the players begin with $x+1$, $x$, and $x-1$ tokens. After three rounds, there will be a net loss of $1$ token per player (they receive two tokens and lose three). Therefore, after $36$ rounds -- or $12$ three-round sets, $A,B$ and $C$ will have $3$, $2$, and $1$ tokens, respectively. After $1$ more round, player $A$ will give away $3$ tokens, leaving them empty-handed, and thus the game will end. We then have there are $36+1=\boxed{\mathrm{(B)}\ 37}$ rounds until the game ends.

Solution 2

Let's bash a few rounds. The amounts are for players $1,2,$ and $3$, respectively.

First round: $15,14,13$ (given) Second round: $12,15,14$ Third round: $13,12,15$ Fourth round: $14,13,12$

We see that after $3$ rounds are played, we have the exact same scenario as the first round but with one token less per player. So, the sequence $1,4,7,10...$ where each of the next members are $3$ greater than the previous one corresponds with the sequence $15,14,13,12...$ where the first sequence represents the round and the second sequence represents the number of tokens player $1$ has. But we note that once player $1$ reaches $3$ coins, the game will end on his next turn as he must give away all his coins. Therefore, we want the $15-3+1=13$th number in the sequence $1,4,7,10...$ which is $\boxed{\mathrm{(B)}\ 37}$.

Solution by franzliszt

See also

2004 AMC 12A (ProblemsAnswer KeyResources)
Preceded by
Problem 6
Followed by
Problem 8
1 2 3 4 5 6 7 8 9 10 11 12 13 14 15 16 17 18 19 20 21 22 23 24 25
All AMC 12 Problems and Solutions
2004 AMC 10A (ProblemsAnswer KeyResources)
Preceded by
Problem 7
Followed by
Problem 9
1 2 3 4 5 6 7 8 9 10 11 12 13 14 15 16 17 18 19 20 21 22 23 24 25
All AMC 10 Problems and Solutions

The problems on this page are copyrighted by the Mathematical Association of America's American Mathematics Competitions. AMC logo.png